Search found 33 matches


Increase in mean value of a set has no direct relation with the increase in median value of the set. Here, we are given the increase in the mean value but we don’t know if the increase is because of change in just 1 value of because of the change in all values. Hence, we can’t determine the chan...

by masoom j negi

Fri Dec 21, 2018 9:08 pm
Forum: Problem Solving
Topic: Median - word problem
Replies: 7
Views: 3277

The set contains 1-6 elements. A. Consider a set {0, 0 ,6}. Here, mean = 2 and median = 0. B. Consider a set { x }. Here, mean = x and median = x C. Consider a set {-1, -1, 5}. Here mean(x) = 1 and median = -1= -x D.Consider a set {1, 1, 1}. Here mean = 1, y = 3 and median = 1 = y/3. E. T is a set o...

by masoom j negi

Fri Dec 21, 2018 9:07 pm
Forum: Problem Solving
Topic: Median of set T
Replies: 9
Views: 3349

The function defines that the terms in Set R are in A.P. with common difference 3. Statement 1.We can’t determine the median of an A.P. with just its first term and Common difference. Last term or the total no of terms is also required. Hence, Insufficient. Statement 2. In A.P. the mean of the ser...

by masoom j negi

Fri Dec 21, 2018 9:06 pm
Forum: Problem Solving
Topic: mgmat 2
Replies: 11
Views: 5887

Any number that will be farthest from the mean of the set will change the standard deviation the most. Here mean of the set is 18/4 = 4.5. Since we have to increase the standard deviation we can’t take any value that is less than the mean. So , 1 and 3 are not acceptable. From 6, 8 and 14, clearly...

by masoom j negi

Fri Dec 21, 2018 9:03 pm
Forum: Problem Solving
Topic: Standard Deviation- Manhattan
Replies: 6
Views: 6387

11Y will be even if Y is even. Y = product of all elements in the set. There is no even no in the set X. So, ‘k’ should be even to make y even. There is only one even prime no i.e. 2. So, k = 2 The set X becomes {2, 3, 7, 11, 17, 19}. Range of the set = highest element – lowest element = 19 â€...

by masoom j negi

Fri Dec 21, 2018 8:58 pm
Forum: Problem Solving
Topic: Range Problem.,...
Replies: 6
Views: 2369

Statement 1. N = ay2 (y is a prime number) N has different value for y = 3, 5, 7 etc. Hence, Insufficient. Statement 2. y4 is a two digit odd integer. y= 3. But we don’t know the relation between ‘n’ and ‘y’ in statement 2. Hence, Insufficient. Statement 1 & 2 together. Using the resul...

by masoom j negi

Fri Dec 21, 2018 8:57 pm
Forum: Data Sufficiency
Topic: Sum of the digits of the positive integer n where n < 99
Replies: 4
Views: 3049

x + y = 77 eq(1) Statement 1. x = y + 1 eq(2) Using eq(1) and eq(2), we get, X = 39 and y = 38. So, xy = 1482. Hence, Sufficient. Statement 2. X = 10a + b Y = 10a + c 10a + b + 10a + c = 77 20a + b + c= 77( where a, b ,c are single digits) This is possible only if (a, b, c) = (3, 8, 9) or (3, 9, 8) ...

by masoom j negi

Fri Dec 21, 2018 8:55 pm
Forum: Data Sufficiency
Topic: The sum of positive integers x and y is 77, what is the
Replies: 3
Views: 946

m + n will be odd if one of them is odd. Statement 1. m = p2 + 4p + 4 = (p + 2)2 So, m is even if p is even and m is odd if p is odd. But we can’t say anything about m+n. Hence, Insufficient. Statement 2. n = p2 + 2m + 1. 2m is even because it is a multiple of 2. P2 + 1 will be even if p is odd an...

by masoom j negi

Fri Dec 21, 2018 8:54 pm
Forum: Data Sufficiency
Topic: If m, n, and p are integers, is m + n odd?
Replies: 4
Views: 1726

Statement 1. 5x – 4y is even i.e. 5x – 4y = 2k Difference of two numbers is even if either both of them are odd or both of them are even. Here, 4y is always even. So, 5x is also even. Since 5x is even, we can say that x is even. Now, xy will be even because the product of an even no with any pos...

by masoom j negi

Fri Dec 21, 2018 8:52 pm
Forum: Data Sufficiency
Topic: Product XY
Replies: 4
Views: 1336

Statement 1. p/4 is a prime no. i.e. p = 4k where k is a prime no. So, p could be anything like 8, 12, 20,……… Hence, Insufficient. Statement 2. P = 3a P could be any positive integer. Hence, Insufficient. Statement 1 & 2 together. P = 4k where k is a prime no and p = 3a So, k = 3. Hence, P...

by masoom j negi

Fri Dec 21, 2018 8:51 pm
Forum: Data Sufficiency
Topic: If p is a positive integer...
Replies: 8
Views: 5892

There is only 1 even prime no i.e. 2.
So y = 4k and x = 4
Then xy = 44k will have a unit digit of 6 because even powers of 4 always have unit digit 6.

by masoom j negi

Fri Dec 21, 2018 8:48 pm
Forum: Problem Solving
Topic: x^y
Replies: 9
Views: 2426

Let d = a.bcdef 100th digit of a.bcdef is c Statement 1. 10d = ab.cdef So, 10th digit of ab.cdef = c = 7 Hence, 100th digit of d = 10th digit of 10d = 7 > 5. Hence, Sufficient. Statement 2. d/10 = 0.abcdef 1000th digit of d/10 = c = 7. So, 100th digit of d = 1000th digit of d/10 = 7>5. Hence, Suffic...

by masoom j negi

Fri Dec 21, 2018 8:45 pm
Forum: Data Sufficiency
Topic: Is the hundredths digit of the decimal d greater than 5?
Replies: 5
Views: 4466

Statement 1. The distance between r and 0 is 3 times the distance between m and 0. r = 3m or -3m. Since we don’t know the distance of ‘m’ from 0, we can’t find the value of ‘r’. Hence, Insufficient. Statement 2. 12 is halfway between m and r. This gives, m + r = 2 x 12 = 24 (m, r) could ...

by masoom j negi

Fri Dec 21, 2018 8:42 pm
Forum: Data Sufficiency
Topic: If m and r are two numbers on a number line, what is the
Replies: 4
Views: 954

Statement 1. If x is rounded to nearest hundredth digit, then 10 – x = 8.56 i.e. x = 1.44 This is possible for (a , b) = (3, >5) and (4 , <5). Hence, Insufficient. Statement 2. If x is rounded to nearest thousandth digit, 10 – x= 8.564 i.e. x = 1.436 This is only possible when a = 3 and b = 5. H...

by masoom j negi

Fri Dec 21, 2018 8:35 pm
Forum: Data Sufficiency
Topic: 1.4ab5
Replies: 6
Views: 4974

answer

Each digit of G is halved to make H. So, H is half of G. So, G + H = 2H + H = 3H So, the sum should be divisible by 3. Thus, C and E can’t be the answer because they are not divisible by 3. Now consider 153 = 102 + 51 It can’t be the value because G should be a 2-digit no. 150 = 100 + 50 It also...

by masoom j negi

Fri Dec 21, 2018 8:34 pm
Forum: Problem Solving
Topic: Sum of two digit number G+H
Replies: 7
Views: 3255